Cambio de sentido en el espacio profundo

Mientras escribía un juego físicamente realista ("Asteroid Defender"), surgió una pregunta física sobre si Diag.1 o Diag.2 o Diag.3 representan correctamente la realidad.

En el espacio profundo (lejos de otros cuerpos celestes), un asteroide perfectamente esférico de masa mse mueve en línea recta con velocidad V 0 relativo al punto C(punto rojo). Su movimiento es constante y uniforme ya que no actúan fuerzas sobre él.

El asteroide tiene una densidad uniforme por lo que su Centro de Masa (CoM) coincide con su centro geométrico. El asteroide es rígido y no se deforma al tocarlo o empujarlo. El asteroide NO gira sobre su CoM. Los rectángulos de color verde pálido que aparecen en el asteroide visualizan la falta de giro del asteroide. Esto se representa a veces t-1y t0en los diagramas.

En ese momento, t1un remolcador espacial maniobrable (empujador espacial para los lectores europeos) aplica una fuerza F 1 a la superficie del asteroide en el punto P1(pequeño punto amarillo) a través de una placa de empuje rígida y plana, que está montada frente al remolcador espacial (línea azul gruesa). Este vector de fuerza se encuentra en una línea que conecta el punto 'P1' y el CoM, por lo que es incapaz de hacer que el asteroide gire alrededor de su CoM.

A medida que avanza el tiempo, el remolcador espacial varía continuamente la dirección de la fuerza aplicada de tal manera que hace que el asteroide atraviese una trayectoria semicircular (giro en U) de radio rcentrado alrededor del punto C. La magnitud de esta fuerza permanece constante durante todo el giro en U, solo su dirección cambia continuamente.
En todo momento, los vectores de fuerza aplicados se encuentran en líneas que conectan el CoM con los puntos en los que la placa de empuje toca la superficie del asteroide (p. ej.: P1en t1, P2en t2, P3en t3, P4en t4, P5ent5). La placa de empuje NO se desliza sobre la superficie del asteroide y no lo hace girar sobre su CoM; la placa de empuje solo empuja al asteroide. Esto se representa en los diagramas a veces desde t1hasta t5.

Una vez que el asteroide completa los 180 grados del giro en U, el remolcador se desconecta y permite que el asteroide se aleje en línea recta a la velocidad V 0 que es paralela pero opuesta a la aproximación inicial. La energía cinética del asteroide antes y después del cambio de sentido es la misma. El asteroide no gira sobre su CoM cuando parte. Esto se representa a veces t6y t7en los diagramas.

PREGUNTA : ¿Qué diagrama representa correctamente la realidad en este escenario?

Justifique por qué un diagrama representa correctamente la realidad y los demás, no.

Diagnóstico 1, representa las líneas ( P1_CoM, ... P5_CoM) que conectan el CoM del asteroide y los puntos en los que la placa de empuje toca la superficie del asteroide ( P1en t1, ... P5en t5), siempre pasando por el centro del giro en U ( punto C). Los vectores ( F 1 , ... F 5 ) se encuentran en estas líneas. Zoom para más detalles. ingrese la descripción de la imagen aquíDiagnóstico 2 y Diag.3 representan las líneas ( P1_CoM, ... P5_CoM) que conectan el CoM del asteroide y los puntos en los que la placa de empuje toca la superficie del asteroide ( P1en t1, ... P5en t5), pasando por los puntos ( Q1, ... Q5), respectivamente, que NO coinciden con el punto C.
En otras palabras: las líneas ( P1_Q1, ... P5_Q5) en las que se encuentran los vectores de fuerza ( F 1 , ... F 5 ), pase a cierta distancia xdel punto C.
Zoom para más detalles. ingrese la descripción de la imagen aquíZoom para más detalles.ingrese la descripción de la imagen aquí

La línea discontinua roja P0_Q0es solo una línea auxiliar que pasa a través del CoM en t1ya través del CoM en el t5punto ya través C. Esta línea no se puede ver sin hacer zoom.

-------------- EDITAR ----------------
Surgió una pregunta en los comentarios a la respuesta de Kamil, si es posible tener una suma de dos vectores A + B tal que la magnitud de esta suma es la misma que la magnitud del vector A ¿solo?
La respuesta es "Sí", pero eso solo es posible cuando el ángulo entre estos dos vectores es >90º y <270º. Vea la prueba formal aquí: https://imgur.com/LELihq9

Otra EDICIÓN: en respuesta a la objeción planteada por Luke Pritchett en los comentarios a continuación, estoy vinculando una respuesta relevante a su objeción: Prevención del giro de asteroides mientras empuja

¿No hay un montón de juegos y diseño de juegos?
Sí, lo hay, pero esta es una cuestión de física. La solicitud no determina su naturaleza.
"La placa de empuje NO se desliza sobre la superficie del asteroide y no lo hace girar sobre su CoM: la placa de empuje solo empuja el asteroide". Estoy bastante seguro de que esto es inconsistente. Para que el empuje no haga girar el asteroide, la fuerza siempre debe apuntar a través de CoM al punto c. Esto significa que el punto de aplicación tiene que girar también en la línea de CoM al punto c, que gira alrededor. Para que esto suceda, la placa tiene que deslizarse a lo largo del exterior del asteroide o el asteroide tiene que girar a la misma velocidad que gira c.
@Luke: Hay una tercera solución. La placa de empuje puede "rodar" relativamente sobre la superficie del asteroide... y esto es exactamente lo que muestran los diagramas. Además, no puedo estar de acuerdo con usted en que: "... Para que la placa de empuje no gire el asteroide, la fuerza siempre debe apuntar a través de CoM al punto C": no es necesario señalar el punto para evitar el giro, solo Capuntar a CoM es necesario para la prevención de giros. Puedo abrir otra pregunta al respecto si ayuda.
La fuerza debe apuntar a c para que el asteroide se mueva en un círculo alrededor de c a una velocidad constante. La fuerza debe apuntar al CoM para que el planeta no gire. Por lo tanto, la fuerza debe apuntar a través de ambos. Sin embargo, no consideré rodar. Eso hace que el Diagrama 1 sea correcto.
Entonces, ¿no hay fricción aquí y, por lo tanto, la fuerza de contacto siempre es perpendicular a la paleta?
ja72: Sí, el vector de fuerza siempre es perpendicular a la placa de empuje y siempre se encuentra en la línea Px_Qx, que pasa por el CoM del asteroide. En otras palabras: el vector de fuerza siempre "apunta a" el CoM.
Como nota al margen, creo que un juego como este sería muy divertido. Intentar controlar la trayectoria de un asteroide masivo solo con una fuerza de contacto (con fricción).

Respuestas (4)

Para obtener una trayectoria semicircular, la aceleración transversal debe ser distinta de cero y constante. Es bastante, simple. Si el asteroide se mueve con velocidad v , y una aceleración transversal constante de a = a T se aplica, entonces el asteroide va en curva con un radio de curvatura igual a r = v 2 / a T . La tasa de barrido va a ser ω = a T / v . La velocidad de salida es v , ya que no hay aceleración longitudinal para acelerar o desacelerar el asteroide.

Esto corresponde al Diagrama 1.

Los diagramas 2 y 3 son incorrectos porque el asteroide no va a atravesar una trayectoria semicircular. Ambos son subconjuntos del problema general, donde la línea de acción tiene un brazo de momento d desde el centro instantáneo de rotación (punto C ). Para el Diagrama 2, d > 0 y para el Diagrama 3 d < 0 . Por supuesto, el Diagrama 1 es d = 0 .

Teniendo en cuenta el ángulo de avance θ formado por d al otro lado de r (el radio de curvatura) la aceleración a se descompone en dos componentes

(1) a T = a porque θ a L = a pecado θ

La trigonometría del problema es tal que d = r pecado θ

bosquejo

Las ecuaciones de movimiento son:

(2) v ˙ = a pecado θ v 2 r = a porque θ

La solución de lo anterior en cada instante es

r = d 2 + ( v 2 a ) 2 v ˙ = a 2 d v 4 + a 2 d 2

lo que significa que el radio depende de la velocidad, y la velocidad sigue cambiando de forma no lineal dependiendo del signo de d . Así, la curvatura de la trayectoria cambia con el tiempo haciendo que el asteroide siga una forma espiral .

¿Cómo concilia la suma de una velocidad v_a(causada por la aceleración a) perpendicular a la velocidad inicial v(que existía antes de esta suma) que da como resultado una suma vectorial v_sumque, según esta demostración , debe tener una magnitud mayor que la velocidad inicial v?
@GeorgeRobinson: es bien sabido que una aceleración perpendicular solo cambia la dirección de la velocidad y no su magnitud.
¿Por qué? Durante cualquier intervalo de tiempo distinto de cero, ¿una aceleración no cambia siempre la magnitud del componente de velocidad que se encuentra en la misma línea que el vector de aceleración (o vector de fuerza)? ...como en este ejemplo
@GeorgeRobinson nunca hay un componente tangente de aceleración y, por lo tanto, la velocidad no cambia. Los comentarios no son un buen lugar para probar esto. Haga una nueva pregunta si no me cree, específica para la aceleración perpendicular.
Me gustaría crear una nueva pregunta sobre la aceleración perpendicular. ¿Podrías proponer el título de esta pregunta para que no tergiverse tus palabras?
Esta pregunta ha sido respondida anteriormente, por lo que no hay necesidad de una nueva pregunta.

En cualquier momento, el componente de fuerza en la línea de (tangente a) la velocidad momentánea cambia la magnitud de la velocidad (es decir, la rapidez), pero no la dirección; la componente de fuerza perpendicular (normal) a la línea de la velocidad momentánea cambia la dirección de la velocidad, pero no su magnitud.

En diag. 1 la fuerza siempre es perpendicular a la línea de la velocidad momentánea, por lo que la velocidad permanece V 0 .

En diag. 2 siempre hay un componente de fuerza contra la velocidad; esto reduce la velocidad, por lo que no se puede V 0 al final de la maniobra.

En diag. 3 siempre hay un componente de fuerza que se suma a la velocidad, por lo que no puede ser V 0 al final de la maniobra.

En cualquier caso, el asteroide puede moverse a lo largo del semicírculo, pero 2 y 3 requieren que el remolcador espacial cambie gradualmente la magnitud de la componente perpendicular de la fuerza, no solo la dirección. Esto se debe a que la componente perpendicular que mantendría una masa metro en la trayectoria circular dada con el radio r depende de la velocidad v :

F pag = metro v 2 r

Creo que puede ser posible mantener constante la magnitud de la fuerza en los casos 2 y 3. El componente perpendicular no constante requeriría un componente tangente no constante, por lo que la magnitud general podría permanecer constante. Aún así, el componente de tangente distinto de cero reduciría (diag. 2) o aumentaría (diag. 3) la velocidad con el tiempo.

De los tres diagramas solo el primero te puede dar V 0 .


Tenga en cuenta que el giro en U en el espacio es un desperdicio de combustible. Si el remolcador espacial simplemente aplicara fuerza hacia la izquierda, eventualmente podría detener el asteroide y luego acelerarlo a V 0 . Los aviones en la atmósfera realizan giros en U a lo largo de semicírculos porque es muy fácil obtener fuerzas normales de la aerodinámica; además, necesitan mantener la velocidad para no detenerse. En el espacio, a menos que necesite una trayectoria específica, simplemente empuje hacia la izquierda el tiempo suficiente para cambiar V 0 a V 0 .

@GeorgeRobinson Si ambos vectores son perpendiculares a la velocidad, se encuentran en un plano perpendicular a la velocidad, por lo que su suma se encuentra en el mismo plano, por lo que la suma también es perpendicular a la velocidad.
¿A qué vectores te refieres? Si hablamos de suma vec, hablemos de sumar vectores de velocidad a vectores de velocidad, o vectores de fuerza a vectores de fuerza. Es obvio que la fuerza causa la aceleración (a=F/m), pero la aceleración no es lo mismo que la velocidad... aunque es la tasa de cambio de la misma. Escribiste: "... cambia la dirección de la velocidad, pero no su magnitud". - ¿Por qué no la magnitud, también? Cuando lanzo una piedra horizontalmente, la atracción gravitacional perpendicular provoca una trayectoria parabólica, en la que cambia tanto la magnitud como la dirección de la velocidad de la piedra.
@GeorgeRobinson, entonces, ¿dónde está la gravedad que mencionas para la roca? Dices que estás en el espacio profundo, por lo que la roca viajará en línea recta.
@Mike: El ejemplo de la roca fue solo una ilustración de que la suma de otro vector perpendicular cambia la dirección y la magnitud de la suma vectorial. En mi Publicación Original no hay gravedad. Sin embargo, el escenario parabólico diferente, aunque relacionado, se puede lograr con un propulsor en lugar de la gravedad (y actuar como un lanzamiento de roca horizontal). Ver: imgur.com/9dwtEPu
NOTA: Cómo en este ejemplo, la aparición de la aceleración en la dirección Y hace que las velocidades V1, ...V8 se sumen a la perpendicular V0, lo que cambia AMBOS la magnitud y la dirección de la suma vectorial resultante VT. Todo sin gravedad. Ver: imgur.com/9dwtEPu
@GeorgeRobinson En este ejemplo, la fuerza (siempre en la dirección Y) no siempre es perpendicular a la velocidad momentánea . Cuando no es perpendicular, existe una componente tangente.
Pero en PRIMERA INSTANCIA el vector agregado ES perpendicular a V0, pero AMBOS la magnitud de la suma es mayor que la magnitud de V0 Y la dirección de la suma es diferente de la dirección de V0. Sin embargo, en su respuesta escribe que el sumando perpendicular " ... cambia la dirección de la velocidad, pero no su magnitud " y eso simplemente no es cierto como se ilustra en ese ejemplo. Sí, puede suceder que el resultado de la suma de vectores tenga una dirección diferente pero no una magnitud, pero solo bajo las condiciones especiales que se mencionan en esta prueba: imgur.com/LELihq9
@GeorgeRobinson Tienes que pensar en términos de d V , no Δ V . En lugar de esto, imagine un empuje constante y una parábola completa. La rapidez (magnitud de la velocidad) tiene su mínimo en el vértice ("parte superior") de la parábola. Mínimo, y cambia suavemente, por lo que su derivada es exactamente cero en este momento. Este es también el único momento en el que la fuerza (empuje) es perpendicular a la velocidad momentánea. No es coincidencia. En este mismo momento la fuerza no cambia la velocidad, solo cambia la dirección de la velocidad.
@Kamil: en la parte superior de esa parábola, la velocidad total es igual a la velocidad horizontal porque la velocidad vertical es cero. En el siguiente caso, la velocidad vertical se vuelve mayor que cero, por lo que la magnitud de la velocidad total aumenta Y su dirección cambia (mientras que la velocidad horizontal no se ve afectada).
@GeorgeRobinson Debajo de una de las otras respuestas, el comentario es "debe considerar cambios de tiempo infinitesimales". No puedo estar más de acuerdo. Si tiene problemas con esto en el caso de nuestra parábola completa, puede ser útil si considera el momento justo antes de la parte superior y el momento justo después, de modo que la parte superior esté exactamente en el medio. En estos dos momentos la magnitud de la velocidad es la misma, la dirección es diferente. La velocidad disminuye antes de la cima, aumenta después de la cima. La cima es el punto mismo donde la velocidad ni disminuye ni aumenta.
@Kamil: el cambio de tiempo infinitesmal es sinónimo de intervalo de tiempo infinitesmal. Infinitesmal es infinitamente pequeño, pero aún mayor que cero. El intervalo de tiempo no puede ser cero porque sin tiempo no hay movimiento. El movimiento en una instancia no es físico; solo es posible en matemáticas abstractas.
@Kamil: Además, el intervalo de tiempo de duración cero parece una contradicción de QM, donde las magnitudes de energía no pueden retroceder infinita y continuamente hacia cero, pero las magnitudes de distancia y tiempo, que las crean, sí pueden. No tienen la Hora de Planck sin una razón.
@GeorgeRobinson Etiquetó la mecánica newtoniana , así que deje fuera la mecánica cuántica. Newton (aparte de Leibniz) desarrolló el cálculo infinitesimal (¡matemáticas abstractas!) exactamente para lidiar con problemas como este en la mecánica newtoniana .

Un objeto con un centro de masa que orbita un punto en una trayectoria circular en el radio r tiene posición vectorial

X ( t ) = r ( porque θ ( t ) , pecado θ ( t ) )
y por lo tanto debe experimentar la fuerza neta
F norte mi t = metro r θ ˙ 2 ( porque θ , pecado θ ) + metro r θ ¨ ( pecado θ , porque θ )
que tiene magnitud
| F norte mi t | = metro r θ ˙ 4 + θ ¨ 2

Para que la magnitud de la fuerza sea constante debemos tener

F F ˙ = 0
ω ˙ ( 2 ω 3 + ω ¨ ) = 0
dónde ω = θ ˙ es la velocidad angular. Hay dos soluciones: ω ˙ = 0 y 2 ω 3 + ω ¨ = 0 . La segunda solución no funciona porque si ω > 0 entonces ω ¨ < 0 , pero eso significaría que el objeto no podría salir del camino semicircular a la misma velocidad con la que comenzó. Esto significa que el objeto debe recorrer el semicírculo a una velocidad constante, con ω ˙ = 0 .

Mirando la ecuación de la fuerza neta vemos que si θ ¨ = 0 , la fuerza siempre apunta al centro del círculo. Y finalmente, si el objeto no debe girar mientras orbita, la fuerza también debe apuntar al centro de masa del objeto. Entonces, si el objeto viaja a una velocidad constante, su Diagrama 1 es la única respuesta correcta.

¿Cómo reconcilia su respuesta con la prueba en imgur.com/LELihq9 ? En Diag.1, la F1 inicial es perpendicular a V0, por lo que solo puede hacer que se agreguen componentes de velocidad perpendicular a V0. Según la demostración, la MAGNITUD del vector suma de V0 + otro vector velocidad, no puede ser igual a la magnitud de V0 cuando el ángulo entre los sumandos es de 90°. Si desea ver qué sucede cuando se agrega otro vector de velocidad a V0 a 90 grados, vea este escenario diferente, aunque relacionado, en: imgur.com/9dwtEPu
El problema con esa prueba es que no usa cambios infinitesimales en la velocidad. Si queremos saber si v tiene magnitud constante en todo momento necesitamos tomar la derivada de | v | = v v . La derivada de eso es d | v | d t = a v | v | , por lo que la velocidad sólo permanece constante cuando a v = 0 . De nuevo, no estamos sumando vectores perpendiculares a v , estamos sumando vectores infinitesimales perpendiculares a v . No hay
Pensé que las reglas de la suma de vectores no se ven afectadas por el tamaño de sus sumandos. De todos modos, ¿cómo puede la adición de vectores infinitamente pequeños, que son PERPENDICULARES a V0, disminuir (... y eventualmente incluso invertir) a (-V0)?
Porque agregas un nuevo infinitesimal en cada instante. Al sumar muchos infinitesimales en cada instante, puede obtener cualquier cambio que desee durante un período de tiempo. Debe considerarlo de esta manera porque tiene una fuerza que cambia constantemente y, por lo tanto, debe considerar cambios de tiempo infinitesimales.
Considere dos vectores de velocidad v A , B con la misma magnitud y ángulo θ entre ellos. El ángulo Δ v = v B v A hace con v A Esta determinado por Δ v v A = v B v A | v A | 2 = | v A | 2 ( porque θ 1 ) . Como consideramos vectores de velocidad que están muy cerca uno del otro en la trayectoria circular, tenemos θ 0 y entonces porque θ 1 0 y Δ v v A 0 .
... pero es esto realista? ¿Los intervalos de tiempo entre las adiciones de vectores realmente van por debajo del tiempo de Planck y se aproximan a cero? A partir de ese análisis, deduzco que usar incluso intervalos ridículamente cortos como el tiempo de Planck produce un resultado diferente que acercarse a intervalos de tiempo de duración cero. Recuerde, estoy interesado en una respuesta que represente con precisión la realidad... no matemáticas abstractas. PD: ¿La pregunta original es realmente tan mala que no merece votos a favor?
Esta es la matemática básica subyacente de la física clásica, en uso desde la época de Newton. Si cree que las Leyes de Newton describen correctamente la realidad, entonces cree que el cálculo describe la realidad con precisión. Dicho de otra manera, si tiene una función de aceleración que cambia constantemente en el tiempo, entonces intenta calcular el movimiento usando solo pasos de tiempo discretos Δ t siempre tendrá un error porque ignora cómo cambió la aceleración entre t y t + Δ t .
Lo pensé y parece que el movimiento no puede ocurrir físicamente en un intervalo de tiempo cero (un instante). Eso solo es posible en matemáticas abstractas. OIA, en física: Sin tiempo = Sin movimiento. Además, parece una contradicción de QM, donde las magnitudes de energía no pueden retroceder infinita y continuamente hacia cero, pero las magnitudes de distancia y tiempo, que las crean, sí pueden.
¿Es posible el movimiento en 0,1 s? ¿Es posible en 0.0001s? ¿Qué tal 0.000000000000001s? O 0.00000000000000000000000000000000000000000001s? ¿Estás planteando la hipótesis de que hay un tiempo más pequeño posible más allá del cual el cambio no es posible? Si es así, ¿dónde está su evidencia para esa hipótesis? En 600 años de física nadie ha encontrado aún ninguna evidencia física por un tiempo más pequeño. Toda la física clásica y moderna se basa en la noción de considerar cambios en intervalos de tiempo arbitrariamente pequeños, que es la idea de la que estoy hablando aquí. Nadie está afirmando que el movimiento ocurre en poco tiempo.
Siempre que el intervalo de tiempo que se considere sea mayor que cero, no tengo ninguna objeción, aunque el Tiempo de Planck PUEDE presentar algún límite en la duración. Lo que realmente me opongo es que las reglas de la suma de vectores se vean afectadas por el tamaño de sus sumandos.
Nadie dijo que las reglas de la suma de vectores cambian cuando cambia el tamaño. Cuando tomas el vector de velocidad de un objeto que se mueve en un círculo en el tiempo t y luego a la hora t + Δ t entonces el vector aceleración a = ( v ( t + Δ t ) v ( t ) ) / Δ t es un vector que es muy, muy cercano a la perpendicular a v cuando Δ t es muy, muy pequeño. Cuanto más pequeño haces Δ t , cuanto más cerca a v ( t ) es a cero. Por esto tiene sentido decir que la aceleración en cualquier instante es perpendicular a la velocidad.

Para invertir la dirección de la nave sin asistencia orbital, la forma más eficiente de combustible sería disparar los propulsores exactamente en la dirección opuesta a la que viaja, hasta que la nave se detenga por completo y luego comience a retroceder. los diagramas mostrados rotarían la nave pero no revertirían eficientemente su curso. En el diagrama uno podría invertir su aproximado si los propulsores se dispararan continuamente en t3 hasta que la nave se detuviera por completo y luego llegara a la velocidad opuesta deseada. Simplemente rotar un proyectil no revertirá su grueso. Para rotar una embarcación de manera eficiente, solo necesita un encendido descentrado para comenzar a girar y luego un encendido igual y opuesto para detener su giro en el punto deseado.

Gracias por la respuesta, pero la eficiencia del combustible del cohete es irrelevante para este problema. La trayectoria semicircular es obligatoria.